Last visit was: 25 Apr 2024, 17:26 It is currently 25 Apr 2024, 17:26

Close
GMAT Club Daily Prep
Thank you for using the timer - this advanced tool can estimate your performance and suggest more practice questions. We have subscribed you to Daily Prep Questions via email.

Customized
for You

we will pick new questions that match your level based on your Timer History

Track
Your Progress

every week, we’ll send you an estimated GMAT score based on your performance

Practice
Pays

we will pick new questions that match your level based on your Timer History
Not interested in getting valuable practice questions and articles delivered to your email? No problem, unsubscribe here.
Close
Request Expert Reply
Confirm Cancel
User avatar
Senior Manager
Senior Manager
Joined: 10 Oct 2005
Posts: 273
Own Kudos [?]: 503 [0]
Given Kudos: 0
Location: US
Send PM
User avatar
Director
Director
Joined: 21 Aug 2006
Posts: 546
Own Kudos [?]: 70 [0]
Given Kudos: 0
Send PM
User avatar
Manager
Manager
Joined: 02 Aug 2006
Posts: 125
Own Kudos [?]: 3284 [0]
Given Kudos: 0
Location: Taipei
Send PM
User avatar
Manager
Manager
Joined: 11 Jul 2006
Posts: 201
Own Kudos [?]: 78 [0]
Given Kudos: 0
Location: TX
Send PM
Re: RC Jeniffer and BRad [#permalink]
D

We have to prove Brad's reasoning that there could be some other explanation to be false .

Its not E because it is one of the other reasons hence strengthens his argument.

Jeneffer says the decrease in video rents rentals is due to the opening of Videorama store which not only rents but also sells videos cheaply(First and only video store that rents and sells).
Brads argument covers just the rentals not the sale of videos. D strenthens Jennifers argument that opening of Videorama stores could have led to more people buying videos rather than renting videos from other stores.

Reduction in sales of other stores appox. = Sales in Rentals at Videorama (4000) + Sales of videos (From D)
User avatar
Manager
Manager
Joined: 25 Sep 2006
Posts: 93
Own Kudos [?]: 15 [0]
Given Kudos: 0
Send PM
Re: RC Jeniffer and BRad [#permalink]
The answer must tie something to buying video. And it mustn't weakens Jennifer's claim.

So, either A or D
But it cannot be A, because that only accounts for at most 8000 declining rental. Wheres the other 2000 ?
Hence, strengtening Brad's reason .

D



Archived Topic
Hi there,
This topic has been closed and archived due to inactivity or violation of community quality standards. No more replies are possible here.
Where to now? Join ongoing discussions on thousands of quality questions in our Reading Comprehension (RC) Forum
Still interested in this question? Check out the "Best Topics" block above for a better discussion on this exact question, as well as several more related questions.
Thank you for understanding, and happy exploring!
GMAT Club Bot
Re: RC Jeniffer and BRad [#permalink]
Moderators:
GMAT Club Verbal Expert
6921 posts
GMAT Club Verbal Expert
238 posts
GRE Forum Moderator
13961 posts

Powered by phpBB © phpBB Group | Emoji artwork provided by EmojiOne